Your-Doctor
Multiple Choice Questions (MCQ)


Quiz Categories Click to expand

Category: Q&A Medicine--->Cardiology
Page: 12

Question 56# Print Question

A 32-year-old woman presents to the hospital with chest pain. She states that the pain is sharp, worse with deep inspiration, and is somewhat relieved by leaning forward. She is febrile and a friction rub is heard on examination. An ECG is shown below.

The patient is offered treatment but refuses all medications. Which of the following is the most common complication if this disease remains untreated?

A. Cardiac tamponade
B. Recurrent pericarditis
C. Constrictive pericarditis
D. Ventricular free wall rupture
E. Valvular insufficiency


Question 57# Print Question

A young man with Marfan syndrome is brought in by paramedics with sharp chest pain that radiates to his back. Shortly after the pain started, he developed left-sided weakness. The chest pain progressed and an ambulance was called. The initial blood pressure recorded by paramedics on the right arm was 78/54 mmHg, but a reading from the left arm shows 114/74 mmHg. An ECG is performed and shows ST elevations in the anterolateral leads. There is widening of the mediastinum on chest x-ray.

Which of the following is the most likely underlying diagnosis?

A. Myocardial infarction
B. Aortic dissection
C. Stroke
D. Acute aortic regurgitation


Question 58# Print Question

A 62-year-old Caucasian man with a history of diabetes and a myocardial infarction 2 years ago is hospitalized for acute decompensated heart failure. He is diuresed with improvement in symptoms and is discharged on appropriate medications. He is seen in clinic 4 months later with complaints of worsening symptoms over the last month. Previously he became mildly short of breath after significant exertion, but he now reports severe shortness of breath after walking only 50 m. He is asymptomatic only at rest. His current medications include aspirin, captopril, carvedilol, furosemide, atorvastatin, omega-3 fatty acids, and pantoprazole. An ECG performed in the office shows Q waves in leads V3 and V4, with a normal QRS duration. An echocardiogram performed 1 week ago showed an ejection fraction of 35%.

The patient has been encouraged to quit smoking and drinking alcohol, and to eat a low-sodium diet. What additional therapy is recommended at this time?

A. Losartan
B. Spironolactone
C. Cardiac resynchronization therapy
D. Hydralazine and isosorbide dinitrate
E. None of the above


Question 59# Print Question

A 42-year-old man presents to the hospital with worsening leg swelling and exertional dyspnea. He denies any recent illness, fevers, weight loss, cough, chest pain, or abdominal pain. His medical history is significant for Hodgkin lymphoma at the age of 20, and he has been in remission after treatment with chemotherapy and radiation. He does not smoke or drink alcohol, and he denies any recent travel or new environmental exposures. His vitals are normal, and there is no scleral icterus or cervical lymphadenopathy. The jugular venous pressure is elevated and increases further during inspiration. Heart sounds are distant with no S3, and the lungs are clear bilaterally. There is hepatomegaly and pitting edema of the lower extremities. A chest x-ray shows a normal heart border without any pulmonary infiltrates. An echocardiogram is performed that shows normal wall thickness with no significant pericardial effusion.

Which of the following is the most likely diagnosis?

A. Pulmonary hypertension
B. Dilated cardiomyopathy
C. Restrictive cardiomyopathy
D. Constrictive pericarditis


Question 60# Print Question

A 73-year-old man with severe aortic stenosis presents to the hospital after passing a moderate amount of blood per rectum. Over the past few years, he has noticed easy bruising and frequent nose bleeds. He has no history of a bleeding disorder, and has no family history of such conditions. He does not take any medications. His vitals are normal, and the examination is unremarkable except for the murmur of aortic stenosis. Laboratory values are drawn and show a hemoglobin level of 11 g/dL and a normal prothrombin time (PT) and partial thromboplastin time (PTT). Factor VIII levels are decreased.

What is the most likely explanation for the episodes of bleeding in this patient?

A. Clotting factor deficiency
B. Platelet deficiency
C. Defect in platelet adhesion
D. Inhibition of antithrombin III
E. Vitamin K deficiency




Category: Q&A Medicine--->Cardiology
Page: 12 of 12